La distancia entre la potencia de 3 y la mayor potencia de 2

Para algún entero positivo k 1 , el mayor índice de bits de 3 k es dado por

metro k registro 2 3

la distancia entre 3 k y 2 metro Se puede escribir como

3 k 2 metro a k 2 metro
dónde 0 < a k < 1 .

Mis preguntas son:

  1. ¿Es cierto que a k 10 C , dónde C es una constante positiva. ¿Cómo puedo probar o refutar esto?
  2. Si la viñeta 1 es verdadera, ¿cómo estimo la constante C ?

Como nota al margen, he calculado numéricamente para k hasta 10.000. Parece que el valor de a k mantenido en un rango estable en ( 0 , 1 ) , y podría ser tan pequeño como O ( 10 6 ) .

Creo que esto ( mathoverflow.net/questions/116840/… ) prueba que no existe tal C
@Arthur Realmente no hay una razón obvia. C definitivamente podría ser no enteros, creo. Editaré y actualizaré mi pregunta para eliminar este límite.
Los valores más pequeños de a será cuando metro / k es una fracción continua impar de registro 2 3.
@Exodd Gracias por el enlace. Sí, prueba que el límite inferior de a depende de metro . Sin embargo, la estimación de la constante en ese enlace no coincide correctamente con lo que observé numéricamente, si es que no me he perdido nada.
Desde a depende de k . deberías escribirlo como a k
Puede que le guste esta pregunta sobre las representaciones racionales de registro 2 ( 3 ) math.stackexchange.com/q/3981318/207316
@ThomasAndrews Reedité para actualizar a -> a k .

Respuestas (2)

Cuando metro k es una fracción continua impar para registro 2 3 entonces:

0 registro 2 3 metro k < 1 k 2

Entonces

3 k = 2 k registro 2 3 < 2 metro + 1 k

Entonces:

3 k 2 metro < 2 metro + 1 / k 2 metro = 2 metro ( 2 1 / k 1 )

Entonces: a = a k 2 1 / k 1. Podemos hacer eso arbitrariamente pequeño.

Las fracciones continuas son infinitas y la mitad de ellas serán impares. (Los términos de fracciones continuas pares te dan ejemplos donde 2 metro > 3 k pero la diferencia es pequeña.)

por ejemplo, cuando k = 12 , metro = 19 , a k .0136 . Cuando k = 53 , metro = 84 , entonces a k 0.00209.


Cuando metro / k es una fracción continua par para registro 2 3 , entonces

2 metro > 3 k > 2 metro 1
entonces:

3 k = 2 k registro 2 3 > 2 metro 1 k
y:
3 k 2 metro 1 > 2 metro 1 ( 2 1 1 / k 1 )

Entonces para metro / k una fracción continua par, como 8 / 5 o sesenta y cinco / 41 , usted obtiene a k 1.

Entonces hay valores de a k arbitrariamente cerca de 0 , y a k arbitrariamente cerca de 1.

No estoy siguiendo la primera desigualdad. 0 registro 2 3 metro k < 1 k 2 . No estoy al tanto de la expansión en fracciones continuas impares para yo o gramo 2 3 , cualquiera. Cuando comprobé sus valores numéricamente. parece que cuando k yo o gramo 2 3 tiene la fracción menor que 0.5 , el obtenido metro / k es lo que llamas la fracción continua impar para yo o gramo 2 3 . ¿Podría explicar un poco cómo se puede obtener la primera desigualdad?
Las fracciones continuas de un número irracional α son una sucesion de numeros racionales pag norte / q norte con | α pag norte . / q norte | < 1 / q norte 2 . Cuando norte es impar, pag norte / q norte < α , cuando norte es par, entonces pag norte / q norte > α . Hay mucho que leer sobre fracciones continuas; estas no son típicas metro , k pero solo un conjunto especial de casos donde obtenemos "buenas" aproximaciones de registro 2 3. @J.Doe
En estos casos, k registro 2 3 estará "cerca" de un número entero, no solo tendrá partes fraccionarias < 0.5. El punto es que siempre podemos encontrar más grande k con la parte fraccionaria de k registro 2 3 menos que 1 k .
Tengo una pregunta más. En el resultado final de la fracción continua impar para yo o gramo 2 3 , a saber, a = a k 2 1 / k 1. Aquí sabemos que no existe el límite inferior para a k . Sin embargo, ¿hay alguna forma de demostrar que el 2 1 / k 1 , o algo similar, es qué tan rápido el valor más pequeño de a k 0 ? Realmente aprecio tu respuesta.
En la última pregunta, no hay realmente una manera. Es posible que registro 2 3 metro / k < 1 / k 3 o incluso menor para valores infinitos de k . @J.Doe
Por favor, carguen con mi ignorancia sobre este tema. ¿Es la existencia de infinitos valores de k eso " yo o gramo 2 3 metro / k < 1 / k 3 o incluso más pequeño" un resultado probado en alguna parte?
Dije "es posible". Es casi seguro que es desconocido.

No hay norte tal que a k Esta abajo 1 / 3 para todos k > norte .

para suponer a k < ( 1 / 3 ) para algunos k . Entonces 2 metro < 3 k < ( 4 / 3 ) 2 metro . Multiplicar por 3 Llegar ( 3 / 2 ) 2 metro + 1 < 3 k + 1 < 2 metro + 2 forzando el valor siguiente, a k + 1 , exceder 1 / 2 .

mmm interesante. Pero aquí hay un ejemplo numérico que parece contradecir. k = 31867 , metro = 50508 , a k = 0.00000726 .
@ J.Doe Esta respuesta es sobre todo k . Puede haber pequeños ejemplos individuales y esto seguiría siendo cierto.
Oscar, espero que mi edición esté bien.
Simplemente use el algoritmo de Euclides para encontrar múltiplos de log_2(3) que sean arbitrariamente cercanos a un número entero.